Download as pdf or txt
Download as pdf or txt
You are on page 1of 44

PHYSICS

EXERCISE 1
1. If an electron is not deflected in passing through a certain space, can we be sure that there is no
magnetic field in that region ?
2. A rectangular current loop is in an arbitrary orientation in an external magnetic field. Is any work
required to rotate the loop about an axis perpendicular to its plane ?
3. Consmic rays are charged particles that strike the atmosphere from some external source. It is found
that more low-energy cosmic rays reach the earth at the north and south magnetic poles than at the
magnetic poles than at the magnetic equator. Why is this so ?
4. Can a charged particle entering a uniform magnetic field normally from outside complete a circle?
5. A very strong current is made to flow for a short time through a solenoid. Will there be any change in
this length and diameter ? Explain.
6. The work done in turning a current loop end-for-end in an external field of strength B is 2mB, where
m is the magnetic moment of the loop. Does this hold no matter what the original orientation of the loop
was ?
7. What is the direction of the magnetic field at points a, b and c due to the current element shown in the
figure?
a

E F i
b
c

8. Particles 1, 2 and 3 follow the paths shown in the figure as they pass through the magnetic field there.
What can one conclude about each particle ?

× × × × × ×
× × × × × ×

× 1× × × × ×
× × × × × ×
2
3
× × × × × ×

9. What is the work done in turning a current loop of area A carrying a current I through an angle q in a
magnetic field B in its own plane ?
10. Will the density of a current be constant throughout the cross-section of a cylindrical conductor ?
11. In electronics, wires that carry equal and opposite currents are often twisted together. Why ?
12. The horse-shoe magnet in a moving coil galvanometer is provided with cylindrical pole-pieces. Why?
13. What is the fundamental difference between the current constituted by electrons in a metallic wire and
the current constituted by an electron beam ?

38
PHYSICS

EXERCISE 8
2
1. The negatively charged disc in figure is rotated clock-wise. What is the direction of the magnetic field
at point A in the plane of the disc?

(A) Into the page (B) Out of the page


(C) Up the page (D) Down the page.
2. Two similar coaxial coils, separated by some distance, carry the same current I but in opposite direc-
tions. The magnitude of the magnetic field B at a point on the axis at the mid point of the line joining the
centre is :
(A) zero (B) the same as that produced by one coil
(C) twice that produced by one coil (D) half of that produced by one coil.
3. In a coaxial, straight cable, the central conductor and the outer conductor carrry equal currents in
opposite directions. The magnetic field is zero.
(A) outside the cable (B) inside the inner conductor
(C) inside the outer conductor (D) in between the two conductors.
4. A particle moves in a circular path of diameter 1.0 m under the action of magnetic field of 0.40 Tesla.
An electric field of 200 V/m makes the path of particle straight. Find the charge / mass ratio of the
particle
(A) 2.5 ´ 105 C/kg (B) 2 ´ 105 C/kg (C) 3.5 ´ 105 C/kg (D) 3 ´ 105 C/kg .
5. Two thin long parallel wires separated by a distance b are carrying a current i amp. each. The magni-
tude of the force per unit length exerted by one wire on the other is
μ 0i 2 μ 0i 2 μ 0i μ 0i
(A) (B) (C) (D) .
b2 2 πb 2 πb πb 2
6. Three long straight parallel wires, carrying current, are arranged as shown in figure. The force experienced
by a 25 cm length of wire C is :
D C G

3cm 2cm

30A 10A 20A


–3 –3
(A) 10 N (B) 2.5×10 N (C) zero (D) 1.5×10–3N
7. For the circuit shown in figure, the direction and magnitude of the force on PQR is :

(A) zero (B) ILB out of the page


(C) ILB/2 into the page (D) ILB into the page

39
PHYSICS
8. A proton, a deutron and an alpha particle with the same kinetic energy enter a region of uniform
magnetic field, moving at right angles to B. The radii of their circular paths are in the ratio :
(A) 1 : 1 : 1 (B) 1 : 2 : 1 (C) 1 : 2 : 2 (D) 1 : 2 : 2
9. In a Bohr model of hydrogen atom, the electron circulates around the nucleus in the path of radius
r at a frequency v revolution s–1. The equivalent dipole moment and magnetic induction at centre
of orbit is
m 0 ev 2 m 0 ev ev m 0 ev 2 m0 v
(A) ev pr , (B) evpr ,
2
(C) , (D) evr ,
2pr 2r pr 2
2r 2re
10. Two infinite plates carry j ampere of current out of the page per unit width of the plate as shown.
BP and BQ represent magnitude of field at points P and Q respectively.

(A) BP = 0 (B) BP = m0j/2 (C) BQ = 0 (D) BQ = m0j/2


11. A long, straight, non-conducting string, painted with a charge density of 40 m c / m , is pulled along
its length at a speed of 300 m/s The magnetic field at a normal distance of 5mm from the moving
string is
(A) 4.8 ´10-7 T (B) zero (C) ¥ (D) Cannot be found
12. Two short magnets of equal dipole moments M are fastened perpendicularly at their centres (figure).
the magnitude of the magnetic field at a distance d from the centre outside the plane of paper on the
bisector of the right angle is -

µ0 M µ0 2M µ 0 2 2M
(A) (B) (C) (D) none of these
4p d 3 4p
d3 4p d 3
13. For what value of r magnetic field due to the circular current
carrying wire at the point P on axis is maximum ?
a a
(A) r = (B) r =
2 2
(C) r = 2 a (D) r = 0
14. Four wires each of length 2 metres are bent into four loops P , Q , R and S and then suspended
into uniform magnetic field (which is parallel to the plane of each loop) . If same current is
passed in each loop, then which of the following statements is correct ?

(A) torque on loop P will be the highest (B) torque on loop Q will be the highest
(C) torque on loop R will be the highest (D) torque on loop S will be the highest
40
PHYSICS

m
15. The dimension of where m is permeability of medium and e is permittivity of medium is same as
e
(A) Resistance (B) Inductance (C) Capacitance (D) speed
16. Two wires are placed at (2,0,0) and (–2,0,0) carrying equal currents along negative z-axis. Magnetic
field at point (0,2,0) is along
(A) positive x-axis (B) negative y-axis (C) positive y-axis (D) negative z-axis
17. Two long wires placed along x and y axis carries equal currents. An electron is projected with some
initial velocity along the line x = y from the origin. The electron
(A) will be deflected towards x-axis. (B) will be deflected towards y-axis
(C) will describe a circle (D) will not deflect at all
18. Which of the following particle will have minimum frequency of revolution when projected with the
same velocity perpendicular to a magnetic field?
(A) electron (B) proton (C) He+ (D) Li+.
19. A particle is projected in a plane perpendicular to a uniform magnetic field. The area bounded by the
path described by the particle is proportional to
(A) the velocity (B) the momentum (C) the kinetic energy (D) none of these.

41
PHYSICS

EXERCISE 3
8
1. A circular loop of radius r carrying a current i is held at the centre of another circular loop of radius
R (>>r) carrying a current I. The plane of the smaller loop makes an angle of 30º with that of the larger
loop. If the smaller loop is held fixed in this position by applying a single force at a point on its
periphery, what would be the minimum magnitude of this force ?
2. A charge of 3.14 × 10–6 C is distributed uniformly over a circular ring of radius 20.0 cm. The ring
rotates about its axis with an angular velocity of 60.0 rad/s. Find the ratio of the electric field to the
magnetic field at a point on the axis at a distance of 5.00 cm from the centre.
3. A circular coil of 200 turns has a radius of 10cm and carries a current of 2.0A. (a) Find the magnitude
r
of the magnetic field B at the centre of the coil. (b) At what distance from the centre along the axis of
the coil will the field B drop to half its value at the centre? (3 4 = 1.5874..)
4. Two parallel wires separated by a distance of 10cm carry currents of 10A and 40A along the same
direction. Where should a third current be placed so that it experiences no magnetic force ?
5. A long wire carrying a current i is bent to form a plane angle a. Find the magnetic field B at a point on
the bisector of this angle situated at a distance x from the vertex.
6. Find the magnetic field B at the centre of a rectangular loop of length l and width b, carrying a
current i.
7. Four long, straight wires, each carrying a current of 5.0A, are placed in a plane as shown in figure. The
points of intersection form a square of side 5.0cm.
(a) Find the magnetic field at the centre P of the square.
(b) Q1, Q2, Q3 and Q4 are points situated on the diagonals of the square and at a distance from P that
is equal to the length of the diagonal of the square. Find the magnetic fields at these points.
Q1 Q2

Q4 Q3

8. Figure shows two parallel wires separated by a distance of 2.0 cm and carrying equal currents of 10A
along opposite directions. Find the magnitude of the magnetic field B at the points A1, A2 and A3 .
A3

2cm

A1 A2
1cm

2cm 2cm

9. An electron emitted by a heated cathode and accelerated through a potential difference of 2.0 kV,
enters a region with uniform magnetic field of 0.15 T. Determine the trajectory of the electron if the
field (a) is transverse to its initial velocity, (b) makes an angle 300 with the initial velocity.

42
PHYSICS

10. A 1.15 kg copper rod rests on two horizontal rails 95.0 cm apart and carries a current of 53.2 A from
one rail to the other. The coefficient of static friction is 0.58. Find the smallest magnetic field (not
necessary vertical) that would cause the bar to slide.
11. A 3.0 cm wire carrying a current of 10A is placed inside a solenoid perpendicular to its axis. The
magnetic field inside the solenoid is given to be 0.27 T. What is the magnetic force on the wire ?
12. A straight horizontal conducting rod of length 0.45 m and mass 60 g is suspended by two vertical wires
at its ends. A current of 5.0 A is set up in the rod through the wires.
(A) What magnetic field should be set up normal to the conductor in order that the tension in the
wires is zero ?
(B) What will be the total tension in the wires if the direction of current is reversed keeping the
magnetic field same as before ? (Ignore the mass of the wires.) g = 9.8 ms-2.
13. A uniform magnetic field of 3000 G is established along the positive z-direction. A rectangular loop of
sides 10 cm and 5 cm carries a current of 12 A. What is the torque on the loop in the different cases
shown in Fig. ? What is the force in each case ? Which case corresponds to stable equilibrium ?
z z

B I B
I
y y

x x
(a) (b)

z z

B B

y y
300
x x
(c) (d)

z z

B B

y y
I I

x x
(e) (f)

14. A circular coil of 20 turns and radius 10 cm is placed in a uniform magnetic field of 0.10 T normal to the
plane of the coil. If the current in the coil is 5.0 A, what is the
(A) Total torque on the coil,
(B) Total force on the coil,
(C) Average force on each electron in the coil due to the magnetic field ?
(The coil is made of copper wire of cross-sectional area 10-5 m2,and the free electron density in
copper is given to be about 1029 m-3.)

43
PHYSICS
15. 4 long wires each carrying current I as shown in figure are placed at the point A, B, C and D. Find the
magnitude and direction of :

(a) magnetic field at the centre of the square


(b) force per meter acting on wire at point D.
16. A ring of mass m and radius r is rotated in magnetic field B which is perpendicular to the plane of the
loop with constant angular velocity w0. Find the net ampere force on the ring and the tension developed
in the ring if there is a current i in the ring. Current and rotation both are clockwise.
x x x x x w0 x x x
x x xi x x x x x
0
x x x x x x x x
x x x x x x x x
x x x x x x x x
x x x x x x x x
x x x x x x x x
x x x x x x x x

44
PHYSICS

EXERCISE 48
1. A uniform magnetic field of magnitude 0.20 T exists in space from east to west. With what speed
should a particle of mass 0.010 g and having a charge 1.0 ´ 10-5 C be projected from south to north so
that it moves with a uniform velocity
(A) 48 m/s (B) 49 m/s (C) 47 m/s (D) 50 m/s
2. A potential difference of 500 V is applied across a parallel plate capacitor. The separation between the
plates is 2×10-3 m. The plates of the capacitor are vertical. An electron is projected vertically upwards
between the plates with a velocity of 105 m/s and it moves undeflected between the plates. The
magnetic field acting perpendicular to the electric field has magnitude of
(A) 0 Wb/m2 (B) 2.5 Wb/m2 (C) 3.0 Wb/m2 (D) 5.0 Wb/m2.
3. Two thin long parallel wires separated by a distance b are carrying a current i amp. each. The magni-
tude of the force per unit length exerted by one wire on the other is
μ 0i 2 μ 0i 2 μ 0i μ 0i
(A) (B) (C) (D) .
b2 2 πb 2 πb πb 2
4. A charge q is moving with a velocity vr 1 = 1 î m / s at a point in a magnetic field and experiences a
r r
force F1 = q ( - ˆj + k̂ ) N. If the charge is moving with a velocity v 2 = 1 ˆj m / s at the same point, it
r r
experiences a force F2 = q ( î - k̂ ) N. The magnetic induction B at that point is :
(A) ( î + ˆj + k̂ ) Wb / m 2 (B) ( î - ˆj + k̂ ) Wb / m 2
(C) (- î + ˆj - k̂ ) Wb / m 2 (D) ( î + ˆj - k̂ ) Wb / m 2
5. Three identical coils carrying a current i each and having a radius a are arranged such that their centres
coincide and their planes are mutually perpendicular. Magnitude of the magnetic field at the centre is

m 0i 3m 0i 3 m 0i
(A) (B) (C) (D) zero
2a 2a 2a
6. A current I flows in a long straight wire with cross section having the form of a half circular ring
of radius R. The magnetic field induction at the point O is B.

m0I m I mI mI
(A) B = (B) B = 0 (C) B = 20 (D) B = 02
4R 4pR pR 2p R
7. A current i flows in the network shown in the figure. Resulting magnetic induction at point P is

m 0i 2m 0i m 0i m 0i
(A) (B) (C) (D)
4pa 8pa 8 2pa 2 pa
45
PHYSICS
8. A current carrying wire is placed in the grooves of an insulating semi circular disc of radius R, as
shown. The current enters at point A and leaves from point B. The magnetic field at point D, has
magntiude :

m0 I m0 I 3m 0 I 3m 0 I
(A) (B) (C) (D)
8pR 3 4pR 3 4pR 2pR
9. A long horizontal wire carries a current I = 1 A . A charged particle of mass m = 1 mg moves
parallel to the wire with constant velocity v = 10 m/s . The distance of the charge from the wire is
r = 1 cm . The magnitude of the charge is :
(A) 5 mC (B) 50 mC (C) 500 mC (D) 5 C
10. Figure shows a conducting square loop of side a which is placed in a plane perpendicular to a uniform
magnetic field B. Points A and D are connected to a battery which sends a current i. The magnetic
force on the loop is
× × × × × ×D i ×
C
× × × × × × ×
B
× × × × × × ×
× × × × × × ×
× × × × × × ×
A E
× × × × × × ×
i
(A) zero (B) 2 2 aiB (C) 2 aiB (D) 2 aiB
æ a a ö
11. Co-ordinates of four corners of a square loop are A º (0,0,0) , B º (0,0, a ) , C º ç - , , a ÷ and
è 2 2 ø
æ a a ö
Dºç , ,0 ÷ . A current I is flowing in the loop in ABCDA direction. The magnetic moment of
è 2 2 ø
the loop would be
æ a2 a2 ö æ a2 a2 ö æ a2 a2 ö æ a2 a2 ö
ç
(A) ç ĵ + k̂ ÷ I ç ĵ - î ÷ I ç ĵ + î ÷ I ç ĵ - î ÷ I
2 ÷ø
(B) ç
2 ÷ø
(C) ç
2 ÷ø
(D) ç
è 2 è 2 è 2 è 2 2 ÷ø

12. A wire of mass 100 g carrying a current of 2A towards increasing x is in the form of y = x2
( - 2 m £ x £ +2m ). This wire is placed in a magnetic field B = -0.02k̂ Tesla. The acceleration of the
wire (in m/s2) is :
(A) - 1.6ˆj (B) - 3.2ˆj (C) 1.6 ĵ (D) 2.4ˆj
13. A wire of length L is shaped into a circle and then folded as shown. Magnetic moment of the
frame when a current i is passed through it is :

L2i L2i L2i L2i


(A) (B) (C) (D)
4p 4 2p 4 8p
46
PHYSICS

EXERCISE 5
8
1. A particle of mass m having a charge q enters into a circular region of radius R with velocity v directed
towards the centre. The strength of magnetic field is B. Find the deviation in the path of the particle.

2. In a certain region uniform electric field E and magnetic field B are present in the opposite direction. At
the instant t = 0, a particle of mass m carrying a charge q is given velocity v 0 at an angle q , with the y-
axis, in the yz plane. Find the time after which the speed of the particle would be minimum.
x

z
E B
q

y v0

3. A tightly-wound, long solenoid has n turns per unit length, a radius r and carries a current i. A particle
having charge Q and mass m is projected from a point on the axis in a direction perpendicular to the
axis. What can be the maximum speed for which the particle does not strike the solenoid ?
4. A wire carrying a 12 A current is bent to pass through various corners of a square of side 20 cm as
r
shown in the figure. What is the force on each segment for B = 0.5 k̂T .

1
3
x

4
z

5. A non-conducting thin spherical shell of radius R has uniform surface charge density s . The shell
rotates about a diameter with constant angular velocity w . Calculate
(a) magnetic induction B at the centre of the shell.
(b) magnetic moment of the sphere.
6. Find the current density as a function of distance r from the axis of a radially symmetrical parallel
stream of electrons if the magnetic induction inside the stream varies as B = bra, where b and a are
positive constants.

47
PHYSICS
7. A small charged ball suspended on an inextensible thread of length l moves as a conical pendulum in
a uniform time independent upward magnetic field of induction B. The mass of ball is m, charge is q
and period of revolution T. Determine the radius of the circle in which the ball moves, if the thread is
always stretched.
8. Figureshow saw irering ofradiusaatrightanglesto thegeneraldirection ofaradially sym m etric
divergingm agneticfield.Them agneticfield attheringiseverywhereofthesam em agnitudeB,andits
direction attheringiseverywhereatan angle q with a normal to the plane of the ring. The twisted lead
wires have no effect on the problem. Find the magnitude and direction of the force the field exerts on
the ring if the ring carries a current i as shown in the figure.

9. Figure shows a wooden cylinder with a mass m = 262 g and a length L = 12.7 cm, with N = 13 turns
of wire wrapped around it longitudinally, so that the plane of the wire loop contains the axis of the
cylinder. What is the least current through the loop that will prevent the cylinder from rolling down a
plane inclined at an angle q to the horizontal, in the presence of a vertical, uniform magnetic field of
477 mT, if the plane of the windings is parallel to the inclined plane ?

L
m
i

10. Magnetic induction at point O if the wire carrying a current I has the shape shown in figure is ___.
Given : The radius of the curved part of the wire is equal to R and linear parts of the wire are very long.

11. A conductor BCD in the form of a major arc of a circle of radius r = 10 cm subtending an angle of 2p/
3 at its centre O is joined to two very long parallel conductors AB and DE as shown in figure. The
parallel conductors AB and DE and normal to the plane of BCD. If a current of I = 100 A flows
through the conductors, the magnetic induction at O is .............................

48
PHYSICS
12. A U-shaped wire of mass m and length l is immersed with its two ends in mercury (see figure). The
wire is in a homogeneous field of magnetic induction B. If a charge, that is, a current pulse q = ò i dt ,
is sent through the wire, the wire will jump up. If the wire reaches a height h = 3m, the size of the charge
or current pulse is _________. Assume that the time of the current pulse is very small in comparison
with the time of flight. B = 0.1 Wb/m2, m = 10 gm, l = 20 cm.

13. Four particles follow the paths shown in figure as they pass through the magnetic field there. What
can one conclude about the charge of each particle ?
× × × × × × × ×
× × × × × × × ×
3
× × × × × × × ×
1× × × × × × × ×
× × × × × × × ×4

2× × × × × × × ×
× × × × × × × ×
× × × × × × × ×
14. A beam of electrons whose kinetic energy is K emerges from a thin-foil “window” at the end of
an accelerator tube. There is a metal plate at a distance d from this window and at right angles to
the direction of the emerging beam (see figure) (a) Show that we can prevent the beam from hitting
2mK
the plate if we apply a magnetic field B such that B ³ in which m and e are the electron
e 2d 2
mass and charge.(b) How should B be oriented ?

d
15. A 22.5 eV positron (positively charged electron) is projected into a uniform magnetic field B = 455
µT with its velocity vector making an angle of 60º with B . Find (a) the period, (b) the pitch p,
and (c) the radius r of the helical path. See figure.

B
v p

q
+ r

16. A rigid circular loop of radius 2 m and mass 5p kg lies in the xy plane on a flat table and has a
r
currentIflow ing in it.A tthisparticularplace,theearth’sm agneticfield isB = 3 iˆ + 4 ĵ . How large
ge
must I (in ampere) be before one edge of the loop will lift from table?

49
PHYSICS

EXERCISE 6
8
NEW IIT-JEE PATTERN QUESTIONS

MULTIPLE CHOICE ANSWER TYPE


1. A long, straight wire carries a current along the Z-axis. One can find two points in the X-Y plane such
that
(A) the magnetic fields are equal
(B) the directions of the magnetic fields are the same
(C) the magnitudes of the magnetic fields are equal
(D) the field at one point is opposite to that at the other point.

2. A charged particle is projected in a plane perpendicular to uniform magnetic field. The areal velocity
(area swept per unit time) of the particle is
(A) directly proportional to kinetic energy of particle
(B) directly proportional to momentum of the particle
(C) inversely proportional to magnetic field strength
(D) inversely proportional to charge on particle
3. Consider the following statements regarding a charged particle in a magnetic field. Which of the
statements are true?
(A) Starting with zero velocity, it accelerates in a direction perpendicular to the magnetic field
(B) While deflecting in magnetic field its energy gradually increases
(C) Only the component of magnetic field perpendicular to the direction of motion of the charged
particle is effective in deflecting it
(D) Direction of deflection force on the moving charged particle is perpendicular to its velocity.

4. Two ions have equal masses but one is singly-ionized and the other is double-ionized. They are
projected from the same place in a uniform magnetic field with the same velocity perpendicular to the
field.
(A) Both ions will go along circles of equal radii
(B) The circle described by the singly-ionized charge will have a radius double that of the other circle
(C) The two circles do not touch each other
(D) The two circles touch each other
5. An electron is moving along the positive X-axis. You want to apply a magnetic field for a short time so
that the electron may reverse its direction and move parallel to the negative X-axis. This can be done
by applying the magnetic field along
(A) Y-axis (B) Z-axis (C) Y-axis only (D) Z-axis only
6. A particle of specific charge q/m = p × 10 C/kg is projected from the origin along the positive x-axis
10

with a velocity of 105 m/s in a uniform magnetic field B = –2 × 10–3 tesla.


(A) The time period of revolution is 10–7 s
(B) The radius of the circular path is 10/p mm
(C) The velocity of the particle at t = 1/6 is 10 j m/s
(D) The velocity of the particle at t = 1/6 s is 5(i + 3 j) m/s

50
PHYSICS
7. Two wires carrying equal currents are placed along x and y axis as shown in figure. The resultant
magnetic field is zero in

(A) First quadrant (B) Second quadrant (C) Third quadrant (D) Fourth quadrant
8. Figure shows the path of an electron in a region of uniform magnetic field. The path consists of
two straight sections, each between a pair of uniformly charged plates, and two half circles. The
electric field exists only between the plates.

(A) Plate I of pair A is at higher potential than plate-II, of the same pair.
(B) Plate I of pair B is at higher potential than plate II of the same pair.
(C) Direction of the magnetic field is out of the page [  ]
(D) Direction of the magnetic field is in to the page [ Ä ]
9. An electron is moving on a circular path of radius r with constant angular velocity w as shown in figure.
r
(A) Direction of B at (0, 0, r) is along k̂
r
(B) Direction of B at (0, 0, r) is along - k̂
r
(C) Direction of B at (0, 0, –r) is along k̂
r
(D) Direction of B at (0, 0, 0) is along - k̂
10. A long, straight wire carries a current along the Z-axis one can find two points in the X-Y plane such
that
(A) the magnetic fields are equal
(B) the directions of the magnetic fields are the same
(C) the magnitudes of the magnetic fields are equal
(D) the field at one point is opposite to that at the other point.
11. The magnetic field between two current carrying wires (whose cross section is shown) varies with
distance from one of the wire as shown in graph. Then :

(A) I1 and I2 are in same direction (B) I1 and I2 are in opposite direction
(C) I1 < I2 (D) I1 > I2
12. Figure shows the path of a charged particle that passes through two regions containing uniform
magnetic field of magnitudes B1 and B2. The direction of magnetic field is perpendicular to the
plane of paper at both the regions. Its path in both the regions is a half circle. Which of the
following(s) is/are correct?

51
PHYSICS

(A) particle is negatively charged (B) B1 < B2


r r r
(C) B2 is into the plane of paper (D) the time spent in B1 is same as that in B2
13. An insulated long wire is bent into circular loop of radius 4 cm and has two straight sections.
The loop is in X-Y plane, with centre at origin. The straight section is parallel to Z-axis. The
wire carries a current of 8 A.

(A) The net magnetic field at origin has magnitude of 40 1 + p 2 mT


(B) The resultant magnetic field makes an angle 90° with X-axis.
(C) The resultant magnetic field makes an angle cos–1 p with positive Z-axis.
1 + p2

(D) The resultant magnetic field has a magnitude of 4 1 + p 2 mT


14. A long straight wire carries a current along the X-axis . Consider the points A (0,1,0),
B (0, 1 , 1) , C (1, 0, 1) and D (1, 1, 1) . Which of the following pairs of points will have magnetic
field of same magnitude ?
(A) A and B (B) A and C (C) B and C (D) B and D

REASONING TYPE
15. Statement-1 : A magnetic monopole does not exist.
Statement-2: Electron spin is always found in pair.
16. Statement-1 : When current is passed through a helical spring its natural length decreases.
Statement-2 : When current is passed through a helical spring its time period increases.
17. Statement-1 : No ‘magnetic potential energy’ can be associated with a moving electric test charge
in a magnetic field.
Statement-2 : Potential energy can be defined only for conservative forces.

i a

18. i b

Statement-1 : A messy loop of limp wire is placed on a frictionless table and anchored at points a
and b as shown in figure. If a current i is now passed through the wire, it will try to form a circular loop
Statement-2 : Two wires carrying current in same direction attract each other.

52
PHYSICS
19. A charged particle enters into a region which offers a resistance against its motion and a uniform
magnetic field exists in the region. The particle traces a spiral path as shown.

P Q

Statement - 1 (A) : If the particle is positively charged, the magnetic field must be outward to
the plane of the paper.
Statement - 2 (R) : Force exerted by the magnetic field is always normal to the direction of
motion. Therefore, its angular momentum remains unchanged.

20. Figure shows a bar magnet and a long straight wire W, carrying current P
N S
into the plane of the paper. Point P is the point of intersection of axis of
magnet and line of shortest distance between magnet and wire (line PW).

Statement - 1 (A) : If P is the midpoint of the magnet, magnet experiences no torque and a
force normal to the line of shortest distance (line PW).
Statement - 2 (R) : North pole experiences a force in the direction of the magnetic field and
south pole experiences a force opposite to the magnetic field.
21. A charged particle moves with a velocity v near a wire carrying an electric current. A magnetic force,
therefore, acts on it. If the same particle is seen from a frame moving with velocity v in the same
direction, the charge will be found at rest.
Statement-1 : The magnetic forces will become zero in this frame.
Statement-2 : The magnetic field will become zero in this frame.
22. The free electrons in a conducting wire are in constant thermal motion.
Statement-1 : If such a wire, carrying no current, is placed in a magnetic field, there is no magnetic
force on each free electron
Statement-2 : There is a magnetic forces on the complete wire.
23. Statement-1 : The torque on a current loop is zero if the angle between the positive normal and
magnetic field is either q = 0 or q = 180º.
Statement-2 : The equilibrium is stable in both the orientations.
24. Statement-1 : If a charged particle at rest experiences no electromagnetic force, the electric field
must be zero.
Statement-2 : The magnetic field in this case may or may not be zero.
25. Statement-1 : A charged particle goes undeflected in a region containing electric and magnetic field.
r r r r
It is possible that E || B, v || E
r r r r
Statement-2 : It is also possible that v || B but E is not parallel to B .
26. Statement-1 : If a charged particle projected in a gravity free room deflects, it means that both fields
can not be zero.
Statement-2 : It is possible in such a case that both fields can be non zero.

53
PHYSICS
m0 i
27. Statement-1 : Magnetic field at a distance r from a thin wire carrying a current i is given by B =.
2p r
From this expression B ® µ as r ® 0. On the other hand, magnetic field at distance r ( £ R) due to
m0 i
a wire of radius R and carrying a current i is given by B = r . From this expression B = 0 at
2p R 2
m0 i
r = 0 and B = at r = r. In this case B is never equal to infinite at any point.
2p R 2

m0 i
Statement-2 : In the derivation of B = due to a thin wire, we take some approximations while
2p r
in the second expression we don’t.
28. Statement-1 : A charged particle is accelerated by a potential difference of V volts. It then enters
perpendicularly to a uniform magnetic field. It rotates in a circle. Its angular momentum about center is
say L. Now if V is doubled, L also becomes two times.
Statement-2 : If V is doubled, kinetic energy will become two times and therefore, L also becomes
to times.
29. Statement-1 : An electron moving with uniform velocity enters a uniform magnetic field perpendicularly
and then a uniform electric field in the same direction. Nature of paths followed by electron in both the
fields will be same.
Statement-2 : The force applied by magnetic and electric field on electron are in different directions.

LINKED COMPREHENSION TYPE


Write Up-1

æ q ö 10
3

A charged particle with charge to mass ratio ç ÷ = C / kg enters a uniform magnetic field
è m ø 19
r r
B = ( 20î - 30 ĵ + 50k̂ ) T at time t = 0 with velocity V = ( 20 î + 50ˆj - 30 k̂ ) m/s. Assume that magnetic
field exists in large space. (use 2 = 1.4 )
r
30. During the further motion of the particle in the magnetic field, the angle between the magnetic field B
and velocity of the particle
(A) remains constant (B) increases
(C) decreases (D) may increase or decrease

31. The frequency of the revolution of the particle in cycles per second will be

103 104 10 4 10 4
(A) (B) (C) (D)
p 19 p 38 p 19 2p 19

32. The pitch of the helical path of the motion of the particle will be
p p p p
(A) m (B) m (C) m (D) m
100 125 215 250

54
PHYSICS
Write-Up – 2
A rigid circular loop has a radius of 0.20 m and is in the x-y plane. A clockwise current I is carried by
the loop, as shown. The magnitude of the magnetic moment of the loop is 0.75 A-m2. A uniform
external magnetic field, B = 0.20 T in the positive x-direction, is present

33. In figure, the magnitude of the magnetic torque exerted on the loop is closest to
(A) 0.55 N-m (B) 0.15 N-m (C) 0.45 N-m (D) 0.35 N-m
34. In figure the loop is released from rest. The initial motion of the loop is described by
(A) point a moves out of the plane, point c moves into the plane
(B) points a, b, c and d move counterclockwise
(C) point a, b, c and d move clockwise
(D) point c moves out of the plane, point a moves into the plane
35. In figure, an external torque changes the orientation of loop from one of lowest potential energy to one
of highest potential energy. The work done by the external torque is closest to
(A) 0.5 J (B) 0.2 J (C) 0.3 J (D) 0.4 J

MATRIX MATCH TYPE


36. The magnetic induction at the point O for wires carrying a current I with the shape shown in
figures is given. The radius of the curved part of the wire is R , the linear parts of the wire are
very long. Match the correct expression from the given one.

(a) (b) (c) (d)


Column I Column II
m 0I
(A) BA (p) B0 = 1 + ( p + 1) 2
4pR

m 0I
(B) BB (q) B0 = 4 + p2
4pR
mI
(C) BC (r) B 0 = 0 ( p + 4)
4pR
m 0I
(D) BD (s) B0 = 2 + 9p 2 / 4
4pR

55
PHYSICS
37. Column I Column II
(A) Charge at rest experiences a force if (p) E = 0, B = 0

(B) A charge in motion goes undeviated (q) E ¹ 0, B ¹ 0


with same velocity if
(C) A charge in motion goes undeviated (r) E = 0, B ¹ 0
with varying speed if
(D) A charged particle undergoes helical (s) E ¹ 0, B = 0
motion if

38. Magnetic moment of


Column I Column II
qwr 2
(A) a uniformly charged ring rotating (p)
5
uniformly about its axis
qwr 2
(B) a charged particle rotating (q)
4
uniformly about a point
qwr 2
(C) a unifomly charged disk rotating (r)
3
uniformly about its axis
qwr 2
(D) a unifomly charged spherical shell (s)
2
rotating uniformly about one of its
diameter
(E) a unifomly charged sphere (t) qwr 2
rotating uniformly about one of its
diameter

39. An electric charge is projected with velocity V into a region where there exists an electric
field E and a magnetic field B .
With the given combination in Column - I match the path in Column - II .
Column – I Column – II
(A) V ^ B, E || B (p) Circular
(B) V || B, E || B (q) Helical

(C) | V | = 0, E ^ B (r) Cycloidal


(D) V ^ B, E = 0 (s) Straight line
40. Column 1 Column 2
(A) Unit of magnetic field (p) Am2
(B) Unit of magnetic permeability (m0) (q) N/Am
(C) Unit of magnetic flux (f) (r) N/A2
(D) Unit of magnetic dipole moment (s) Nm/A

56
PHYSICS

EXERCISE 8
7
1. State Bio-Savarts law .
r
2. What is the magnitude of the force on a charge q moving with a velocity v in a magnetic field B ?
uur
3. What is the force that a conductor dl carrying a current I experiences when placed in a magnetic field
r
B . What is the direction of the force?

4. Define Si unit of magnetic field.

5. Define ampere in terms of force acting between two infinitely long parallel current carrying
conductors.

6. What is Lorentz force?

7. Under what conditions an electron moving through a magnetic field experiences maximum force ?

8. What is the nature of the magnetic field inside a long solenoid? Write an expression for this magnetic
field.

9. What is the order of the ratio of the electric and magnetic forces between two moving charges?

10. An electron is not deflected while passing through a region. Are you sure that there is no magnetic field
in that region ?

11. Does the time taken by a charged particle to complete one revolution within a uniform magnetic field
depend on :

(a) radius of the path (b) mass of the particle (c) velocity of the particle.

12. How many gauss equal one tesla?

13. A charged particle moving in straight line enters a uniform magnetic field at an angle of 45º. What will
be its path?

14. Write the expression for the magnetic field due to a circular current carry coil at its centre.

15. Explain why two straight, parallel wires, carrying currents in opposite direction repel each other.

16. What are the factors upon which the current sensitivity of a moving coil galvanometer depends?

17. Why are the pole pieces of the magnet mode concave in a moving coil galvanometer?

18. How can we use galvanometer as voltmeters?

19. How can we increase the range of a given ammeter ?

57
PHYSICS
20. What are the limitations of cyclotron?

21. An electron is moving along x axis in a magnetic field which is along y axis. What is the direction of
magnetic force of the electron?

22. Why does the kinetic energy of a charged particle moving a uniform magnetic field remain constant?

23. A current is flowing through a straight thin metallic wire of infinite length. Obtain an expression for the
magnetic field at a distance d from it.

24. Using Biot-savart law, derive an expression for the magnet field at the centre of a current-carrying coil.

25. Derive an expression for the magnetic field due to a current-carrying coil at a point on its axis.

26. A current carrying rectangular loop is placed in a uniform magnetic field such that the normal to its
plane is inclined at an angle Q to the direction of the magnetic field. Derive an expression for the torque
acting on it.

27. Derive an expression for the force acting per unit length between two long straight parallel current
carrying conductors.

28. Explain the construction and working of a moving coil galvanometer with the help of a diagram.

29. Explain by drawing a labelled diagram how a positively charged particle is accelerated in a cyclotron.

30. Derive an expression for the radius of the circular path followed by a charged particle crossing a
uniform magnetic field normally.

58
PHYSICS

EXERCISE 8
1. If in a circular coil A of radius R, current I is flowing and in another coil B of radius 2R a current 2I is
flow ing,then theratio ofthem agneticfieldsB A and BB, produced by them will be[AIEEE-2002]
(A) 1 (B) 2 (C) 1/2 (D) 4
2. If a current is passed through a spring then the spring will
(A) expand (B) compress (C) remains same (D) none of these
3. Wires 1 and 2 carrying currents i1 and i2 respectively are inclined at an angle q to each other. What is
the force on a small element dl of wire 2 at a distance of r from wire 1 (as shown in figure) due to the
magnetic field of wire 1? [AIEEE-2002]
1 2

i1 r i2

q dl

m0 m0 m0 m0
(A) i1i 2 dl tan q (B) i1i 2 dl sin q (C) i1i 2 dl cos q (D) i1i 2 dl sin q
2pr 2pr 2pr 4pr
4. The time period of a charged particle undergoing a circular motion in a uniform magnetic field is
independent of its [AIEEE-2002]
(A) speed (B) mass (C) charge (D) magnetic induction
5. r
A particle of mass M and charge Q moving with velocity v describe a circular path of radius R when
subjected to a uniform transverse magnetic field of induction B. The work done by the field when the
particle completes one full circle is [AIEEE-2003]

æ Mv 2 ö
(A) ç R ÷ 2pR (B) zero (C) B Q 2pR (D) B Q v2pR
è ø
6. A particle of charge –16 × 10–18 coulomb moving with velocity 10ms–1 along the x-axis enters a region
where a magnetic field of induction B is along the y-axis, and an electric field of magnitude 104 V/m is
along the negative z-axis. If the charged particle continues moving along the x-axis, the magnitude of B
is : [AIEEE-2003]
(A) 103 Wb/m2 (B) 105 Wb/m2 (C) 1016 Wb/m2 (D) 10–3 Wb/m2
7. The magnetic lines of force inside a bar magnet [AIEEE-2003]
(A) are from north-pole of south-pole of the magnetic
(B) do not exist
(C) depend upon the area of cross-section of the bar magnetic
(D) are from south-pole to north-pole of the magnet
8. A current i ampere flows along an infinitely along straight thin walled tube, then the magnetic induction
at any point inside the tube is [AIEEE-2004]

m 0 2i 2i
(A) Tesla (B) zero (C) infinite (D) Tesla
4p r r
59
PHYSICS
9. A long wire carries a steady current. It is bent into a circle of one turn and the magnetic field at the
centre of the coil is B. It is bent into a circular loop of n turns. The magnetic field at the centre of the coil
will be [AIEEE-2004]
(A) 2nB (B) n2B (C) nB (D) 2n2B
10. The magnetic field due to a current carrying circular loop of radius 3 cm at a point on the axis at a
distance of 4cm from the centre is 54mT . What will be its value of the centre of loop ?
[AIEEE-2004]
(A) 125 mT (B) 150 mT (C) 250 mT (D) 75 mT
11. Two long conductors, separated by a distance d carry current I1 and I2 in the same direction. They
exert a force F on each other. Now the current in one of them is increased to two times and its
direction is reversed. The distance is also increased to 3d. The new value of the force between them
is : [AIEEE-2004]

2F F F
(A) - (B) (C) –2F (D) –
3 3 3
12. The length of a magnet is large compared to its width and breadth. The time period of its oscillation in
a vibration magnetometer is 2s. The magnet is cut along its length into three equal parts and these parts
are then placed on each other with their like poles together. The time period of this combination will be
[AIEEE-2004]

2 2
(A) 2 3 s (B) s (C) 2s (D) s
3 3
13. Two thin, long, parallel wires, separated by a distance ‘d’ carry a current of ‘i’A in the same direction.
They will [AIEEE-2005]
(A) repel each other with a force of m0i2 /(2pd)
(B) attract each other with a force of m0i2 /(2pd)
(C) repel each other with a force of m0i2 /(2pd2)
(D) attract each other with a force of m0i2 /(2pd2)
14. A charged particle of mass m and charge q travels on a circular path of radius r that is perpendicular
to a magnetic field B. The time taken by the particle to complete one revolution is [AIEEE-2005]

2pq 2 B 2pmq 2pm 2pqm


(A) (B) (C) (D)
m B qB m
15. A uniform electric field and a uniform magnetic field are acting along the same direction on a certain
region. If an electron is projected along the direction of the fields with a certain velocity then
(A) its velocity will increase [AIEEE-2005]
(B) its velocity will decrease
(C) it will turn towards left of direction of motion
(D) it will turn towards right of direction of motion

60
PHYSICS
16. In a region, steady and uniform electric and magnetic fields are present. These two fields are parallel
to each other. A charged particle is released from rest in this region. The path of the particle will be a
[AIEEE-2006]
(A) helix (B) straight line (C) ellipse (D) circle
17. A long straight wire of radius a carries a steady current i. The current is uniformly distributed across its
cross section. The ratio of the magnetic field at a/2 and 2a is [AIEEE-2007]
(A) 1/2 (B) 1/4 (C) 4 (D) 1
18. A current I flows along the length of an infinitely long, straight, line walled pipe. Then
(A) the magnetic field at all points inside the pipe is the same, but not zero
(B) the magnetic field is zero only on the axis of the pipe [AIEEE-2007]
(C) the magnetic field is different at different points inside the pipe
(D) the magnetic field at any point inside the pipe is zero
r
19. A charged particle with charge q enters a region of constant, uniform and mutually orthogonal fields E
r r r r
and B with a velocity v perpendicularly to both E and B , and comes out without any change in
r
magnitude or direction of v . Then [AIEEE-2007]
r r r r r r r r
(A) vr = B ´ E / E 2 (B) vr = E ´ B / B2 (C) vr = B ´ E / B2 (D) vr = E ´ B / E 2
20. A charged particle moves through a magnetic field perpendicular to its direction. Then
(A) kinetic energy change but the momentum is constant [AIEEE-2007]
(B) the momentum changes but the kinetic energy is constant
(C) both momentum and kinetic energy of the particle are not constant
(D) both momentum and kinetic energy of the particle are constant
21. Two identical conducting wires AOB and COD are placed at right angles to each other. The wire
AOB carries an electric current I1 and COD carries a current I2. The magnetic field on a point lying at
a distance d from O, in a direction perpendicular to the plane of the wires AOB and COD, will be
given by [AIEEE-2007]

m0 m 0 æ I1 + I 2 ö m0 m0
1/ 2

( I12 + I 22 ) ( I1 + I2 )
1/ 2
(A) (I12 + I 22 ) (B) ç ÷ (C) (D)
2pd 2pd è d ø 2pd 2pd
22. Two parallel, long wires carry currents i1 and i2 with i1 > i2. When the currents are in the same direction,
the magnetic field at a point midway between the wires is 30 mT . If the direction of current is reversed,
the field becomes 90µT. The ratio i1 / i 2 is [AIEEE-2010]
(A) 4 (B) 3 (C) 2 (D) 1
23. A current I flows in an infinitely long wire with cross section in the form of a semicircular ring of radius
R. The magnitude of the magnetic induction along its axis is [AIEEE-2011]
m0 I m0 I m0 I m0 I
(A) (B) (C) (D) 2
2p R
2
2pR 4p R
2
p R
24. Proton, Deuteron and alpha particle of the same kinetic energy are moving in circular trajectories in a
constant magnetic field. The radii of proton, deuteron and alpha particle are respectively rp, rd and
ra.Which one of the following relations is correct? [AIEEE-2012]
(A) ra > rd > rp (B) ra = rd > rp (C) ra = rp = rd (D) ra = rp < rd

61
PHYSICS

EXERCISE 9
8
1. A magnetic needle is kept in a non-uniform magnetic field. It experiences : [1982, 3M]
(A) a force and a torque (B) a force but not a torque
(C) a torque but not a force (D) neither a force nor a torque
2. A conducting circular loop of radius r carries a constant current i. It is placed in a uniform magnetic
r r
field B0 such that B0 is perpendicular to the plane of the loop. The magnetic force acting on the loop
is : [1982, 3M]
r r r
(A) irB0 (B) 2pirB0 (C) zero (D) pirB0
3. A rectangular loop carrying a current i is situated near a long straight wire such that the wire is parallel
to one of the sides of the loop and is in the plane of the loop. If steady current I is established in the wire
as shown in the figure, the loop will : [1985, 2M]
(A) rotate about an axis parallel to the wire
(B) move away from the wire
(C) move towards the wire
(D) remain stationary
4. Two thin long parallel wires separated by a distance ‘b’ are carrying a current ‘i’ ampere each. The
magnitude of the force per unit length exerted by one wire on the other is : [1986, 2M]
m 0i 2 m 0i 2 m 0i m 0i
(A) (B) (C) (D)
b2 2pb 2pb 2pb 2
5. Two particles X and Y having equal charges, after being accelerated through the same potential difference,
enter a region of uniform magnetic field and describe circular paths of radii R1 and R2 respectively. The
ratio of the mass of X to that of Y is : [1988, 2M]
1/2 2
(A) (R1/R2) (B) R2/R1 (C) (R1/R2) (D) R1/R2
6. A battery is connected between two points A and B the circumference of a uniform conducting ring of
radius r and resistance R. One of the arcs AB of the ring subtends an angle q at the centre. The value
of the magnetic induction at the centre due to the current in the ring is -
(A) proportional to (180° – q) (B) inversely proportional to r
(C) zero, only if q = 180° (D) zero for all values of q [JEE - 95, 2]
7. A proton, a deutron and an a-particle having the same kinetic energy are moving in circular trajectories
in a constant magnetic field. If rp , rd & ra denote respectively the radii of the trajectories of these
particles, then : [JEE - 97, 1]
(A) ra = rp < rd (B) ra > rd > rp (C) ra = rd > rp (D) rp = rd = ra
8. Two particles, each of mass m & charge q, are attached to the two ends of a light rigid rod of length
2R. The rod is rotated at constant angular speed about a perpendicular axis passing through its centre.
The ratio of the magnitudes of the magnetic moment of the system & its angular momentum about the
centre of the rod is - [1998, 2M]
q q 2q q
(A) (B) (C) (D)
2m m m pm
9. Select the correct alternative (s) - [1998, 2M]
(i) Two very long, straight, parallel wires carry steady currents I & – I respectively. The distance
between the wires is d. At a certain instant of time, a point charge q is at a point equidistant from the
r
two wires, in the plane of the wires. Its instantaneous velocity v is perpendicular to this plane. The
magnitude of the force due to the magnetic field acting on the charge at this instant is :
µ 0 Iqv µ 0 Iqv 2 µ 0 Iqv
(A) (B) (C) (D) 0 [1998, 2M]
2pd pd pd

62
PHYSICS
(ii) Let [Î0] denote the dimensional formula of the permittivity of the vacuum and [µ0] that of the
permeability of the vacuum. If M = mass, L = length, T = time and I = electric current, then
(A) [Î0] = M–1 L–3 T2 I (B) [Î0] = M—1 L–3 T4 I2
(C) [µ0] = MLT–2 I–2 (D) [µ0] = ML2 T–1 I
10. A charged particle is released from rest in a region of steady and uniform electric and magnetic fields
which are parallel to each other. The particle will move in a - [1999, 2M]
(A) straight line (B) circle (C) helix (D) cycloid
11. A circular loop of radius R, carrying current I, lies in x-y plane with its centre at origin. The total
magnetic flux through x-y plane is :
(A) directly proportional to I (B) directly proportional to R
(C) inversely proportional to R (D) zero
12. An infinitely long conductor PQR is bent to form a right angle as shown. M
A current I flow through PQR. The magnetic field due to this current
at the point M is H1. Now, another infinitely long straight conductor
900
QS, is connected at Q so that the current in PQ remaining unchanged. -µ P I Q 900 S +¥

H1 R
The magnetic field at M is now H2. The ratio H is given by -
2

1 2
(A) (B) 1 (C) (D) 2 [2000, 2M]
2 3
13. An ionized gas contains both positive and negative ions. If it is subjected simultaneously to an electric
field along the + x direction and a magnetic field along the + z direction, then
(A) positive ions deflect towards + y direction and negative ions towards – y direction
(B) all ions deflect towards + y direction [2000, 2M]
(C) all ions deflect towards – y direction
(D) positive ions deflect towards – y direction and negative ions towards + y direction.
14. A particle of charge q and mass m moves in a circular orbit of radius r with angular speed w. The
ratio of the magnitude of its magnetic moment to that of its angular momentum depends on -
(A) w and q (B) w, q and m (C) q and m (D) w and m [2000, 2M]
15. Two long parallel wires are at a distance 2d apart. They carry equal steady currents flowing out of
the plane of the paper as shown. The variation of the magnetic field B along the line ‘XX’ is given by-
[2000, 2M]

(A) (B) (C) (D)

16. A non-planar loop of conducting wire carrying a current I is placed as shown


in figure. Each of the straight sections of the loop is of length 2a. The magnetic
field due to this loop at the point P (a, 0, a) points in the direction -
[2001, 2M]
1 1
(A) (– ˆj + k̂ ) (B) (– ˆj + k̂ + î )
2 3
1 1
(C) ( î + ˆj + k̂ ) (D) ( î + k̂ )
3 2
63
PHYSICS
17. A coil having N turns is wound tightly in the form of a spiral with inner and outer radii a and b respectively.
When a current I passes through the coil, the magnetic field at the centre is - [2001, 2M]

µ 0 Nl 2µ 0 Nl µ0 Nl b µ0IN b
(A) (B) (C) ln (D) ln
b a 2(b - a ) a 2(b - a) a
18. Two particles A and B of masses mA and mB respectively and having the same charge are moving in a
plane. A uniform magnetic field exists perpendicular to this plane. The speeds of the particles are vA
and vB respectively and the trajectories are as shown in the figure. Then - [JEE- 2001 Screening]
A
B

(A) mA vA < mB vB (B) mA vA > mB vB


(C) mA < mB and vA < vB (D) mA = mB and vA = vB
19. A long straight wire along the z-axis carries a current I in the negative z direction. The magnetic vector
r
field B at a point having coordinates (x, y) in the z = 0 plane is - [2002, 3]

æ yî - xˆj ö µ 0 I æç x î + yĵ ö÷ µ 0 I æç xˆj - yî ö÷ æ x î - yˆj ö


µ0Iç 2 ÷ µ0I ç 2 ÷
(A)
2p ç x + y2 ÷ (B)
2 p çè x + y ÷ø
2 2 (C)
2 p çè x + y ÷ø
2 2 (D)
2p ç x + y2 ÷
è ø è ø
20. The magnetic field lines due to a bar magnet are correctly shown in - [2002, 2M]

(A) (B) (C) (D)

21. A particle of mass m and charge q moves with a constant velocity v along the positive x direction. It
enters a region containing a uniform magnetic field B directed along the negative z direction, extending
from x = a to x = b. The minimum value of v required so that the particle can just enter the region x >
b is - [2002, 2M]
(A) q b B/m (B) q ( b – a) B/m (C) q a B/m (D) q (b + a) B/2m
22. For a positively charged particle moving in a x-y plane initially along the x-axis, there is a sudden
change in its path due to the presence of electric and/or magnetic fields beyond P. The curved path is
shown in the x-y plane and is found to be non-circular. Which one of the following combinations is
possible? [2003, 2M]
r r r r
(A) E = 0; B = bˆj + ck̂ (B) E = a î; B = ck̂ + a î
r r r r
(C) E = 0; B = cˆj + bk̂ (D) E = a î; B = ck̂ + bˆj
23. A conducting loop carrying a current I is placed in a uniform magnetic field pointing into the plane of the
paper as shown. The loop will have a tendency to : [2003, 2M]

(A) contract (B) expand


(C) move towards +ve x-axis (D) move towards –ve x-axis

64
PHYSICS
24. A current carrying loop is placed in a uniform magnetic field in four different orientations, I, II, III and
IV, arrange them in the decreasing order of potential energy : [2003, 2M]

(I) (II)

(III) (IV)

(A) I > III > II > IV (B) I > II > III > IV
(C) I > IV > II > III (D) III > IV > I > II

25. An electron moving with a speed u along the positive x-axis at y = 0 enters a region of uniform
r
magnetic field B = -B0 k̂ which exists to the right of y-axis. The electron exits from the region after
sometime with the speed v at co-ordinate y, then : [2004, 2M]

(A) v > u, y < 0 (B) v = u, y > 0 (C) v > u, y > 0 (D) v = u, y < 0
r r
26. A magnetic field B = B0 ĵ exists in the region a < x < 2a and B = -B0 ĵ , in the region 2a < x < 3a,
r
where B0 is a positive constant. A positive point charge moving with a velocity v = v 0 î , where v0 is a
positive constant, enters the magnetic field at x = a. The trajectory of the charge in this region can be
like,
B0

a 2a 3a x

-B 0

z z

2a
(A) a 2a 3a x (B) a 3a x

z z

a 2a 3a
(C) a 2a 3a x (D) x

65
PHYSICS
OBJECTIVE PROBLEMS (more than one option are correct)
27. A proton moving wth a constant velocity passes through a region of space without any change in its
velocity. If E and B represent the electric and magnetic fields respectively, this region of space may
have : [1985, 2M]
(A) E = 0, B = 0 (B) E = 0, B ¹ 0 (C) E ¹ 0, B = 0 (D) E ¹ 0, B ¹ 0
28. H+, He+ and O2+ all having the same kinetic energy pass through a region in which there is a uniform
magnetic field perpendicular to their velocity. The masses of H+, He+ and O2+ are 1 amu, 4 amu and 16
amu respectively. Then : [1994, 2M]
+ 2+
(A) H will be deflected most (B) O will be deflected most
(C) He+ and O2+ will be deflected equally (D) all will be deflected equally
29. The adjacent figure shows lines of a field. It cannot represent [2006]

(A) an electrostatic field (B) an induced electric field


(C) a gravitational field (D) a magnetostatic field
30. The figure shows an infinitely long current carrying wire out of the plane of the paper (shown as a dot
‘ ’). A current carrying loop ABCD is placed as shown in the figure. The loop

(A) experiences no net force


(B) experiences no torque
(C) turns clockwise as seen by an observer located at the dot (‘ ’)
(D) turns anti-clockwise as seen by an observer located at the dot (‘ ’) [2006]
31. A particle of mass m and charge q, moving with velocity V enters Region II normal to the boundary
as shown in the figure. Region II has a uniform magnetic field B perpendicular to the plane of the
paper. The length of the Region II is l. Choose the correct choice(s) [2008]
Figure.
Region I Region II Region III

qlB
(A) The particle enters Region III only if its velocity V >
m
qlB
(B) The particle enters Region III only if its velocity V<
m
qlB
(C) Path length of the particle in Region II is maximum when velocity V =
m
(D) Time spent in Region II is same for any velocity V as long as the particle returns to Region I

66
PHYSICS
REASONING TYPE
32. STATEMENT - 1 [2008]
The sensitivity of a moving coil galvanometer is increased by placing a suitable magnetic material
as a core inside the coil.
and
STATEMENT - 2
Soft iron has a high magnetic permeability and cannot be easily magnetized or demagnetized.
(A) STATEMENT-1 is True, STATEMENT -2 is True; STATEMENT-2 is a correct explanation
for STATEMENT-1
(B) STATEMENT-1 is True, STATEMENT -2 is True; STATEMENT-2 is NOT a correct
explanation for STATEMENT-1
(C) STATEMENT-1 is True, STATEMENT -2 is False
(D) STATEMENT-1 is False, STATEMENT -2 is True
33. Which of the field (s) given in column - II will be produced by a loop mentioned in column - I
Column I Column II [2006]
(A) Dielectric ring having uniform (p) Electric field
charge is stationary
(B) Dielectric ring having uniform (q) Magnetostatic field
charge rotating with constant
angular velocity
(C) A constant current I0 in the (r) Outside the loop time dependent induced
loop electric field
(D) Time varying current I = I0 (s) Magnetic moment in the loop
coswt
34. Column I gives certain situations in which a straight metallic wire of resistance R is used an Column II
gives some resulting effects. Match the statements in Column I with the statements in Column II and
indicate your answer by darkening appropriate bubbles in the 4×4 matrix given in the ORS.
Column I Column II
(A) A charged capacitor is connected (p) A constant current flows through
to the ends of the wire the wire
(B) The wire is moved perpendicular (q) Thermal energy is generated in
to its length with a constant velocity the wire
magnetic field perpendicular to the
plane of motion
(C) The wire is placed in a constant (r) A constant potential difference
electric field that has a direction develops between the ends of
along the length of the wire the wire
(D) A battery of constant emf is (s) Charges of constant magnitude
connected to the ends of the wire appear at the ends of the wire
[2007, 6M]
67
PHYSICS
35. Two wires each carrying a steady current I are shown in four configurations in Column I. Some of the
resulting effects are described in Column II. Match the situations in Column I with the characteristeristic
in Column II and indicate your answer by darkening appropriate bubbles in the 4×4 matrix given in
the ORS.
Column I Column II
(A) Point P is situated
midway between the wires. (p) The magnetic fields (B) at
P due to the currents in the
wires are in the same
direction.

(B) Point P is situated at the mid-point (q) The magnetic field (B) at
of the line joining the centers of the P due to the currents in the
circular wires, which have same radii. wires are in opp. direction.

(C) Point P is situated at the mid-point (r) There is no magnetic


of the line joining the centers of the field at P
circular wires, which have same radii.
P

(D) Point P is situated at the common (s) The wires repel each other.
centre of the wires.

P
[2007, 6M]

36. A thin flexible wire of length L is connected to two adjacent fixed points and carries a current I in the
clockwise direction, as shown in the figure. When the system is put in a uniform magnetic field of
strength B going into the plane of the paper, the wire takes the shape of a circle. The tension in the wire
is [2009-4M]
× × × × × ×
× × × × × ×
× × × × × ×
× × × × × ×
× × × × × ×
× × × × × ×
× × × × × ×
× × × × × ×
IBL IBL IBL
(A) IBL (B) (C) (D)
p 2p 4p
37. The figure shows certain wire segments joined together to form a coplanar loop. The loop is placed in
a perpendicular magnetic field in the direction going into the plane of the figure. The magnitude of the
field increases with time. I1 and I2 are the currents in the segments ab and cd. Then, [2009-4M]

68
PHYSICS
c I2 d
× × × × ×
(A) I1 > I2 a I1 b
(B) I1 < I2 × × × × ×

(C) I1 is in the direction ba and I2 is in the direction cd × × × × ×


× × × × ×
(D) I1 is in the direction ab and I2 is in the direction dc
× × × × ×
38. An electron and a proton are moving on straight parallel paths with same velocity. They enter a semi-
infinite region of uniform magnetic field perpendicular to the velocity. Which of the following statement
(s) is/are true ? [2011. 6M]
(A) They will never come out of the magnetic field region.
(B) They will come out travelling along parallel paths.
(C) The will come out at the same time.
(D) They will come out of different times.
39. Outsider the motion of a positive point charge in a region where there are simultaneous uniform
r r
electric and magnetic fields E = E 0ˆj and B = B0ˆj . At time t = 0, this charge has velocity vr
in the x-y plane, making an angle q with the x-axis. Which of the following option(s) (is/are)
correct for time t > 0 ? [2012]
(A) If q = 0º, the charge moves in a circular path in the x-z plane.
(B) If q = 0º, the charge undergoes helical motion with constant pitch along the y-axis.
(C) If q = 10º, the charge undergoes helical motion with its pitch increasing with time, along
the y-axis.
(D) If q = 90º, the charge undergoes linear but accelerated motion along the y-axis.

40. A cylindrical cavity of diameter a exists inside a cylinder of diameter


2a as shown in the figure. Both the cylinder and the cavity the
infinitely long. A uniform current density J flows along the length. P a
If the magnitude of the magnetic field at the point P is given by
N
m 0 aJ , then the value of N is . [2012] 2a
12

41. A loop carrying current I lies in the x-y plane as shown in the figure. The unit vector k̂ is coming
out of the plane of the paper. The magnetic moment of the current loop is [2012]

æp ö 2 ˆ æp ö 2 ˆ
(A) a 2 Ikˆ (B) ç + 1÷ a Ik (C) - ç + 1÷ a Ik (D) ( 2p + 1) a 2 Ikˆ
è2 ø è2 ø

69
PHYSICS

EXERCISE 10
8
1. A neutron, a proton and an electron and alpha particle enter a region of constant magnetic field with
equal velocities. The magnetic field is along the inward normal to the plane of the paper. The tracks of
the particles are labelled in figure. The electron follows track .............. and the alpha particle follows
track ................................ [1984, 2M]

2. A wire of length L metres carrying a current i amperes is bent in the form of circle. The magnitude of
its magnetic moment is .............. in MKS units. [1987, 2M]
16
3. In a hydrogen atom, the electron moves in an orbit of radius 0.5Å making 10 revolutions per second.
The magnetic moment associated with the orbital motion of the electron is .........
[1988, 2M]
4. The wire loop PQRSP formed by joining two semicircular wires of radii R1 and R2 carries a current I
as shown. The magnitude of the magnetic induction at the centre C is .......... [1988, 2M]

5. A wire ABCDEF (with each side of length L) bent as shown in figure and carrying a current I is placed
in a uniform magnetic induction B parallel to the positive y-axis. The force experienced by the wire is
____________ in the _________ direction. [1990, 2M]

r
6. A metallic block carrying current I is subjected to a uniform magnetic induction as B as shown in
r
figure. The moving charges experience a force F given by .............. which results in the lowering of
the potential of the face .............. Assume the speed of the carries to be V. [1996, 2M]

7. A uniform magnetic field with a slit system as shown in figure is to be used as momentum filter for high-
energy charged particles. With a field B Tesla, it is found that the filter transmits a-particles each of
energy 5.3 MeV. The magnetic field is increased to 2.3 B Tesla and deuterons are passed into the filter.
The energy of each deuteron transmitted by the filter is ................. MeV. [1997C, 1M]

70
PHYSICS

TRUE/FALSE
8. No net force acts on a rectangular coil carrying a steady current when suspended freely in a uniform
magnetic field. [1981, 2M]
9. There is no change in the energy of a charged particle moving in magnetic field although a magnetic
force is acting on it. 1983, 2M]
10. A charged particle enters a region of uniform magnetic field at an angle of 85° to the magnetic line of
force. The path of the particle is a circle. [1983, 2M]
11. An electron and proton are moving with the same kinetic energy along the same direction. When they
pass through a uniform magnetic field perpendicular to the direction of their motion, they describe
circular path of the same radius. [1985, 3M]
SUBJECTIVE
12. A potential difference of 600 volts is applied across the plates of a parallel plate condenser. The
separation between the plates is 3 mm. An electron projected vertically, parallel to the plates, with a
velocity of 2 × 106 m/s moves undeflected between the plates. Find the magnitude and direction of the
magnetic field in the region between the condenser plates. (Neglect the edge effects). (Charge of the
electron = 1.6 × 10–19 C) [1981, 3M]

13. A particle of mass m = 1.6 × 10–27 kg and charge q = 1.6 × 10–19C enters a region of uniform magnetic
field of strength 1 tesla along the direction shown in figure. The speed of the particle is 107 m/s.
[1984, 8M]
(a) The magnetic field is directed along the inward normal to the plane
of the paper. The particle leaves the region of the field at the point F.
Find the distance EF and the angle q.
(b) If the direction of the field is along the outward normal to the plane
of the paper, find the time spent by the particle in the region of the
magnetic field after entering it at E.
14. A beam of protons with a velocity 4 × 105 m/s enters a uniform magnetic field of 0.3 tesla at an angle
of 60° to the magnetic field. Find the radius of the helical path taken by the proton beam. Also find the
pitch of the helix (which is the distance travelled by a proton in the beam parallel to the magnetic field
during one period of rotation). [1986, 6M]
15. Two long straight parallel wires are 2 m apart, perpendicular to the plane of the paper. The wire A
carries a current of 9.6 ampere, directed into the plane of the paper. The wire B carries a current such
that the magnetic field of the induction at the point P, at a distance of 10/11 m from the wire B, is zero.

71
PHYSICS
Find [1987, 7M]
(a) The magnitude and direction of the current at B.
(b) The magnitude of the magnetic field of induction at the point S.
(c) The force per unit length on the wire B.

16. A pair of stationary and infinitely long bent wires are placed in the xy planes as shown in figure. The
segments L and M are along the x-axis. The segments P & Q are parallel to the Y - axis such that
OS = OR = 0.02 m. Find the magnitude and direction of the magnetic induction at the origin O.
i = 10 A [JEE - 89, 6]

17. Two long parallel wires carrying currents 2.5 amps & I amps in the same
direction (directed into the plane of the paper) are held at P & Q
respectively such that they are perpendicular to the plane of paper. The
points P & Q are located at a distance of 5 meters and 2 m respectively
from a collinear point R. (See figure)
(i) An electron moving with a velocity of 4 × 105 m/s along the positive X-direction experiences a
force of magnitude 3.2 × 10–20 N at the point R. Find the value of I.
(ii) Find all the positions at which a third long-parallel wire carrying a current of magnitude 2.5 amps
may be placed so that the magnetic induction at R is zero. [1990, 8M]
18. An electron gun G emits electrons of energy 2 kev travelling in the (+) ve x-direction. The electrons are
required to hit the spot S where GS = 0.1 m r & the line GS makes an angle of 60° with the x-axis, as
shown in figure. A uniform magnetic field B parallel to GS exists in the region outside to electron gun.
Find the minimum value of B needed to make the electron hit S. [1993, 7M]

19. An electron in the ground state of hydrogen atom is revolving in anitclock-wise


direction in a circular orbit of radius R.
(i) Obtain an expression for the orbital magnetic dipole moment of the electron
r
(ii) The atom is placed in a uniform magnetic induction B such that the plane normal of the electron
orbit makes an angle of 30° with the magnetic induction. Find the torque experienced by the
orbiting electron.. [1996, 5M]
20. 3 Infinitely long thin wires each carrying current i in the same direction, are in the x-y plane of a gravity
free space. The central wire is along the y-axis while the other two are along x = ± d.
(i) Find the locus of the points for which the magnetic field B is zero [1997, 5M]
(ii) If the central wire is displaced along the z-direction by a small amount & released, show that it will
execute simple harmonic motion. If the linear density of the wires is l , Find the frequency of oscillation.

72
PHYSICS
21. A particle of mass m & charge q is moving in a region where uniform, constant electric and magnetic
r r r r r
field E & B are present, E & B are parallel to each other. At time t = 0 the velocity v 0 of the particle
r
is perpendicular to E . (assume that its speed is always << c, the speed of light in vacuum). Find the
r r
velocity v of the particle at time t. You must express your answer in terms of t, q, m, the vectors v 0 ,
r r
E & B and their magnitudes v0, E & B . [1998, 8M]
22. A current of 10A flows around a closed path in a circuit which is in the horizontal plane as shown in the
figure. The circuit consists of eight alternating arcs of radii r1 = 0.08 m and r2 = 0.12 m . Each arc
subtends the same angle at the centre.
(i) Find the magnetic field produced by this circuit at the centre.
(ii) An infinitely long straight wire carrying a current of 10A is passing through
the centre of the above circuit vertically with the direction of the current being
into the plane of the circuit. What is the force acting on the wire at the centre
due to the current in the circuit ? What is the force acting on the arc AC and the
straight segment CD due to the current at the centre ? [2001, 5 + 5]
23. A ring of radius R having uniformly distributed charge Q is mounted on a rod suspended by two
identical strings. The tension in strings in equilibrium is T0. Now a vertical magnetic field is switched on
and ring is rotated at constant angular velocity w. Find the maximum w with which the ring can be
rotated if the strings can withstand a maximum tension of 3T0/4. [2003, 4M]

24. A proton and an alpha particle, after being accelerated through same potential diffrerence, enter uniform
magnetic field, the direction of which is perpendicular to their velocities. Find the ratio of radii of the
circular paths of the two particles. [2004, 2M]
25. A moving coil galvanometer experiences torque = ki where i is current. If N coils of area A each and
moment of inertial I is kept in magnetic field B. [2005, 6M]
(a) find k in terms of given parameters.
(b) if for current i deflection is p/2, find out torsional constant of spring.
(c) if a charge Q is passed suddenly through the galvanometer, find out maximum angle of deflection.

26. A long circular tube of length 10m and radius 0.3m carries a current I along its curved surface as
shown . A wire-loop of resistance 0.005 ohm and of radius 0.1m is placed inside the tube with its axis
coinciding with the axis of the tube. The current varies as I = I0cos (300t) where I0 is constant. If the
magnetic moment of the loop is N µ0 I0 sin (300 t), then ‘N’ is [2011, 6M]

73
PHYSICS

EXERCISE 8
11
5 5
Q.1 Two circular coils A and B of radius cm and 5 cm respectively carry current 5 Amp and Amp
2 2
respectively. The plane of B is perpendicular to plane of A and their centres coincide. If the magnetic
5
field at the centre is given by ´ 4p´ 10-5 T . Find the value of N.
N 2
Q.2 What is the magnitude of magnetic field at the centre ‘O’ of loop of radius 2 m made of uniform wire
when a current of 1 amp enters in the loop and taken out of it by two long wires as shown in the figure.

Q.3 If the current carrying wire is in the shape shown in the figure. When the
m 0i é N ù
magnetic induction at point O is ê p + 1ú . Then find the value of N.
4pr ë 2 û
Q.4 An electron moving with a velocity 5 × 106 ms–1 î in the uniform electric field of 5 × 107 Vm–1 ĵ . Find
the magnitude a minimum uniform magnetic field in tesla that will cause the electron to move undeviated
along its original path. (answer in the multiple of 10)
Q.5 A charged particle (charge q, mass m) has velocity v0 at origin in +x direction. In space there is a
uniform magnetic field B in - z direction. If the y coordinate of particle when is crosses y axis is
2mv 0
N ´ qB . Find the value of N.
Q.6 A conducting circular loop of radius r carries a constant current i. It is placed in a uniform magnetic
r r
field B0 such that B0 is perpendicular to the plane of the loop. Find the magnetic force acting on the
loop is
Q.7 Two long straight parallel conductors are separated by a distance of r1 = 5cm and carry currents
i1 = 10 A & i2 = 20 A . What work per unit length of a conductor must be done to increase the
separation between the conductors to r2 = 10 cm if , currents flow in the same direction? (answer
in multiple of 27.6 mJ/m)

Q.8 Q charge is uniformly distributed over the same surface of a right circular cone of
semi-vertical angle q and height h. The cone is uniformly rotated about its axis at
Qw 2
angular velocity w. If associated magnetic dipole moment is h tan 2 q . Find
N
the value of N.

Q.9 A charge particle A of charge q = 2 C has velocity v = 100 m/s. When it passes through
point A and has velocity in the direction shown. The strength of magnetic field at point
B due to this moving charge is (r = 2 m) is 25 × 10–x T. Then find the value of x.

74
PHYSICS
Q.10 Three rings, each having equal radius R, are placed mutually perpendicular to
each other and each having its centre at the origin of co-ordinate system. If
current I is flowing through each ring, then the magnitude of the magnetic field
m0I
at the common centre is N . Find the value of N.
2R

Q.11 Infinite number of straight wires each carrying current I are equally
placed as shown in the figure. Adjacent wires have current in
opposite direction. If the net magnetic field at point P is given by
m0 I ln 4

4p N a . Find the value of N.

Q.12 If the magnetic field at P due to the arrangement shown is m 0i æç N - 1 ö÷ Ä . Find the value of N.
2pd è 2ø

Q.13 Equal current i is flowing in three infinitely long wires along positive x, y and z directions. If the magnetic
m 0i ˆ ˆi
field at a point (0, 0, –a) is given by ( j - ) . Find the value of N.
2pa N

Q.14 A long straight wire, carrying current I, is bent at its midpoint to from an angle
of 45°. Induction of magnetic field at point P, distant R from point of bending is

equal to
( )
N -1 m0 I
. Find the value of N.
4pR
Q.15 A hollow cylinder having infinite length and carrying uniform current per unit length l along the
circumference as shown. Magnetic field inside the cylinder is m0lN.. Find the value of N.

Q.16 A mass spectrometer is a device which select particle of equal mass. An


iron with electric charge q > 0 and mass m starts at rest from a source
S and is accelerated through a potential difference V. It passes through a
r
hole into a region of constant magnetic field B perpendicular to the plane
of the paper as shown in the figure. The particle is deflected by the magnetic
field and emerges through the bottom hole at a distance d from the top hole.
qB2 d 2
If the mass of the particle is . Find the value of N.
N´V
75
PHYSICS

Q.17 A conducting wire bent in the form of a parabola y2 = 2x carries a current i = 2 A as shown in figure.
r
This wire is placed in a uniform magnetic field B = -4 k̂ Tesla. If the magnetic force on the wire is (in
newton) (answer in multiple of 8iˆ )

Q.18 A circular current loop of radius a is placed in a radial field B as shown. If the net force acting on the
loop is NpaIBsinq. Find the value of N.

Q.19 A rectangular coil PQ has 2n turns, an area 2a and carries a current 2I, (refer figure). The plane of the
coil is at 60° to a horizontal uniform magnetic field of flux density B. If the torque on the coil due to
magnetic force is N BnaI cos60°. Find the value of N.

Q.20 Electric charge q is uniformly distributed over a rod of length l. The rod is placed parallel to a long wire
carrying a current i. The separation between the rod and the wire is a. If the force needed to move the
m 0iqv
rod along its length with a uniform velocity v is N pa . Find the value of N.

76
PHYSICS

PP- ANSWER KEY


PP-1

m 0i
Q.1 (D) Q.2 (B) Q.3 6.6 × 10–5 T Q.4
2R

é 1 3 ù
Q.5 (D) Q.6 m0i ê 8R + 8R ú
ë 2 1û

PP-2
Q.1 (C) Q.2 0.12 mT Q.3 (C)
Q.4 2 2B in the plane of drawing from right to left Q.5 (B)
Q.6 (A) Q.7 (C)

PP-3

m0 I m0 I
Q.1 ( p + 2) upward Q.2 ( p - 2) donnward
4pR 4pR

é 3 1 ù
Q.3 m0 I ê + ú donnward Q.4 (C)
ë 8R 2 2a û

m 0i ˆ m 0i
Q.5 i- ( p + 2)kˆ Q.6 (C) Q.7 (A)
4R 8pR

PP-4
Q.1 0,m0 K towards right in the figure, 0 Q.2 2p × 10–3 T

m 0i (r 2 - a 2 )
Q.3 1V Q.4
2p(b 2 - a 2 ) r

m0 Ir m0 I m0 I æ r 2 - b2 ö
Q.5 (a) 2 (b) (c) ç1 - 2 2 ÷ Q.6 (C) Q.7 (A)
2pa 2pa 2pr è c -b ø

m 0 I æ 2r 2 - a 2 ö m 0 I æ 2r 2 + a 2 ö
Q.8 (a) 2pr ç 2 2 ÷ to left (b) 2pr ç 2 2 ÷ towards top of the paper
è 4r - a ø è 4r + a ø

77
PHYSICS
PP-5
Q.1 (A) Q.2 (C) Q.3 (C) equal to g
Q.4 (D) Q.5 (C) Q.6 (C)

Bq -1 æ dqB ö m æ dqB ö
Q.7 (a) t (b) sin ç ÷ (c) sin -1 ç ÷
m è mV ø Bq è mv ø

v0
Q.8 v = v0 cos (B0 a t) i + v0 sin–1 (B0 a t)j, r = [sin (B0a t)i + (1 - cos B0a t) j]
B0 a
qm 0 In
Q.9 (B) Q.10
4r 2 m

PP-6

Q.1 (A) , (B) Q.2 (B) Q.3 (A) Q.4 (C)

Q.5 (C) Q.6 (D) Q.7 (B)

PP-7

Q.1 (C) Q.2 (B), (C) Q.3 (D) Q.4 (D)

mg
Q.5 B0il in positive x direction Q.6 (A)
pIr

78
PHYSICS

ANSWER SHEET
Exercise - 02
1. (A) 2. (A) 3. (A) 4. (A)
5. (B) 6. (C) 7. (A) 8. (D)
9. (B) 10. (A) 11. (A) 12. (B)
13. (D) 14. (D) 15. (A) 16. (A)
17. (D) 18. (D) 19. (C)

Exercise - 03
1. m 0 Iipr / 4R 2. 3 × 1020 : 3200 3. (a) 4p × 10–4 (b) x = ± 2R
m0 I é æ a öù
4. 2cm from 10 A wire 5. ê1 + cos ç ÷ ú
æaö è 2 øû
2px sin ç ÷ ë
è2ø
8m 0il 8m 0
6. 7. (a) zero (b) Q1 = Q3 =  , Q2 = Q4 = 0
pb l + b 2 2
3p

10
8. A1 = 5 × 10–5 T, A2 = 40 × 10–5 T A3 = ´ 10-7 T
2
r
9. (a) Circular trajectory of radius 1.0 mm normal to B
r
(b) helical trajectory of radius 0.5 mm with velocity component 2.3 ´ 107 ms -1 along B .
10. 0.115 T 11. 8.1 × 10–2N; direction of force given by Fleming’s left-hand rule
12. (a) A horizontal magnetic field of magnitude 0.26 T normal to the conductor in such a direction that
Fleming’s left-hand rule gives a magnetic force upwards.
(b) 1.176 N.
r r r r r r
13. Use t = IA ´ B and F = Il ´ B
(a) 1.8 ´ 10-2 N m along - y direction (b) same as in (a)
-2
(c) 1.8 ´ 10 N m along -x direction
(d) 1.8 ´ 10-2 N m at an angle of 240º with the +x direction (e) zero (f) zero
Force is zero in each case. Case (e) corresponds to stable, and case (f) corresponds to unstable
equilibrium.
14. (a) zero (b) zero
(c) force on each electron is evB = IB /(nA) = 5 ´ 10-25 N . Note Answer denotes only the magnetic
force.
m0 I 10 m 0 I 2 r
15. (a) along y-axis, (b) 16. 0, ( mw02 + 2piB)
p a 8 pa 2p

Exercise - 04
1. (B) 2. (B) 3. (B) 4. (A)
5. (C) 6. (C) 7. (B) 8. (B)
9. (B) 10. (D) 11. (B) 12. (C)
13. (B)
79
PHYSICS

Exercise - 05
æ mv ö mv0 sin q m0 Qrni
1. p - 2 tan -1 ç ÷ 2. 3.
è qBR ø qE 2m
4. F1 = 1.2iN
ˆ , F = 1.2ˆjN , F = ( -1.2iˆ - 1.2ˆj)N , F = 1.2ˆjN
2 3 4

2 4 æ bö
5. (a) m 0 swR , (b) pswR 4 6. j(r ) = çç ÷÷(1 + d )µd -1
3 3 è µ0 ø
2
æ mg ö 2p
7. r = l - çç
2
÷ , w=
2 ÷ 8. 2p aiBsinq, out of the page
è qBw + mw ø T

2
æ µ 0 I ö æç 3µ 0 I ö÷
2
µ 0 I æ - 3p ö µI
ç - 1÷ î - 0 k̂ 11. ç ÷ +
è 3R ø çè 4pR ÷ø
9. 1.63 A 10.
4pR è 2 ø 4pR
12. 3.8C 13. 1 : + ; 2 : –; 3 : 0 ; 4 : –.
–6
15. (a)6.077×10 sec(b) 0.11m (c) 0.03m 16. 5

Exercise - 06
1. (B) (C) (D) 2. (A) (C) (D) 3. (C) (D) 4. (B) (D)
5. (A) (B) 6. (A) (B) (C) 7. (A) (C) 8. (A) (B) (C)
r r
9. (B) Direction of B at (0, 0, r) is along - k̂ , (D) Direction of B at (0, 0, 0) is along - k̂
10. (B) (C) (D) 11. (A) 12. (A) (B) 13. (A) (B)
14. (B) (D) 15. [A] 16. [C] 17. [A]
18. [B] 19. [C] 20. [D] 21. [C]
22. [C] 23. [C] 24. [B] 25. [C]
26. [B] 27. [C] 28. [D] 29. [B]
30. (A) 31. (B) 32. (D) 33. (B)
34. (A) 35. (C)
36. A-q , B-p , C-s , D-r 37. A - q,s B - q,p C -qs, D -q, r
38. A - s , B - s , C - q, D - r, E - t 39. A - q , B - s , C - r, D -p
40. A - q , B - r , C - s, D - p

Exercise - 08
1. (A) 2. (B) 3. (C) 4. (A)
5. (C) 6. (A) 7. (D) 8. (B)
9. (B) 10. (C) 11. (A) 12. (B)
13. (B) 14. (C) 15. (B) 16. (B)
17. (D) 18. (D) 19. (B) 20. (B)
21. (C) 22. (C) 23. (D) 24. (D)

Exercise - 09
1. (A) 2. (C) 3. (C) 4. (B)
5. (C) 6. (D) 7. (A) 8. (A)
9. (D) 10. (A) 11. (D) 12. (C)
13. (C) 14. (C) 15. (B) 16. (D)
80
PHYSICS
17. (C) 18. (B) 19. (A) 20. (D)
21. (B) 22. (B) 23. (B) 24. (C)
25. (D) 26. (A) 27. (A, B, D) 28. (A, C)
29. (A, C) 30. (A, C) 31. (A, C, D) 32. (C)
33. (A) - (p), (B) - (p, q), (C) - (q, s), (D) - (r, s) 34. (A)- (q), (B)-r,s, (C)-s, (D)-p,q,r
35. (A)- (q,r), (B)-p (C)-q,r (D)-q, or (A) - (q, r) (B) - (p) (C) - (q, r) (D) - (q, s)
36. (C) 37. (D) 38. (A) (D)
39. (C) (D) 40. (5) 41. (B)

Exercise - 10
Fill in the blanks
L2i
1. D, B 2. 3. 1.26 × 10–23 Am2
4p

m0I æ 1 1 ö
çç - ÷ (perpendicular to paper outwards)
4 è R 1 R 2 ÷ø
4. 5. IlB, positive Z

6. eVBk̂ , ABCD 7. 14.0185


True and False
8. T 9. T 10. F 11. F
Subjective ProblemS
12. 0.1 T (perpendicular to paper inwards)

13. (a) 0.14m, 45°, (b) 4.712 × 10–8 s 14. 1.2 × 10–2 m, 4.37 × 10–2 m

15. (a) 3A, perpendicular to paper outwards, (b) 13 × 10–7 T, (c) 2.88 × 10–6 N/m

16. 10–4 T, perpendicular to paper outwards

17. (a) 4A, (b) At distance 1 m from R to the left or right of it, current is outwards if placed to the left and
inwards if placed to the right of R.
eh ehB r r
18. 4.73 × 10–3 T 19. (i) M = , (ii) t = , perpendicular to both M and B
4pm 8pm
r r
d i m0 r æ qB ö r æq ö r æ qB öæç v 0 ´ B ö÷
20. (a) x = ± , (b) f = 21. v = cosç m t ÷( v 0 ) + ç m t ÷(E) + sinç m t ÷ç B ÷
3 2pd pl è ø è ø è øè ø

22. (a) 6.54 × 10–5 T (Vertically upward or outward normal to the paper)
(b), zero, zero, 8.1×10–6N (inwards)
DT0 1 2BiNA BNAp
23. wmax = 24. 25. (a) k = BNA, (b) K = , (c) Q 26. 6
BQR 2 2 p 2l

Exercise - 11
Q.1 2 Q.2 0 Q.3 3 Q.4 1 Q.5 1
Q.6 0 Q.7 1 Q.8 4 Q.9 7 Q.10 3
Q.11 3 Q.12 1 Q.13 1 Q.14 2 Q.15 1
Q.16 8 Q.17 4 Q.18 2 Q.19 8 Q.20 2
81

You might also like